Diễn Đàn MathScopeDiễn Đàn MathScope
  Diễn Đàn MathScope
Ghi Danh Hỏi/Ðáp Thành Viên Social Groups Lịch Ðánh Dấu Ðã Ðọc

Go Back   Diễn Đàn MathScope > Sơ Cấp > Việt Nam và IMO > 2015

News & Announcements

Ngoài một số quy định đã được nêu trong phần Quy định của Ghi Danh , mọi người tranh thủ bỏ ra 5 phút để đọc thêm một số Quy định sau để khỏi bị treo nick ở MathScope nhé !

* Nội quy MathScope.Org

* Một số quy định chung !

* Quy định về việc viết bài trong diễn đàn MathScope

* Nếu bạn muốn gia nhập đội ngũ BQT thì vui lòng tham gia tại đây

* Những câu hỏi thường gặp

* Về việc viết bài trong Box Đại học và Sau đại học


Trả lời Gởi Ðề Tài Mới
 
Ðiều Chỉnh Xếp Bài
Old 10-07-2015, 03:04 PM   #1
huynhcongbang
Administrator

 
huynhcongbang's Avatar
 
Tham gia ngày: Feb 2009
Đến từ: Ho Chi Minh City
Bài gởi: 2,413
Thanks: 2,165
Thanked 4,188 Times in 1,381 Posts
Gửi tin nhắn qua Yahoo chát tới huynhcongbang
IMO 2015 - Đề thi, lời giải và kết quả đội tuyển

Dưới đây là đề thi Olympic Toán quốc tế 2015 ngày thi thứ 1 mình lấy từ facebook của thầy phó đoàn Việt Nam, TS. Lê Anh Vinh.

ĐỀ THI IMO 2015

Bài 1.
Một tập hợp hữu hạn $S$ các điểm nằm trên mặt phẳng là "cân bằng" nếu như với hai điểm $A,B$ phân biệt thuộc $S$, luôn tồn tại một điểm $C$ thuộc $S$ mà $AC=BC.$ Ta gọi tập hợp $S$ là "không tâm" nếu như với mọi bộ ba điểm $A,B$ và $C$ thuộc $S,$ không tồn tại điểm $P$ thuộc $S$ sao cho $PA=PB=PC.$
a) Chứng minh rằng với mọi số tự nhiên $n\ge 3$, tồn tại một tập hợp cân bằng có $n$ điểm.
b) Tìm tất cả các số tự nhiên $n\ge 3$ sao cho tồn tại một tập vừa cân bằng, vừa không tâm và có $n$ điểm.

Bài 2.
Xác định tất cả bộ ba $(a,b,c)$ các số nguyên dương sao cho các số sau
$ab-c,\text{ }bc-a,\text{ }ca-b$
đều là các lũy thừa của $2.$

Bài 3.
Cho tam giác $ABC$ nhọn với $AB>AC.$ Gọi $\Gamma $ là đường tròn ngoại tiếp tam giác, $H$ là trực tâm của tam giác và $F$ là chân đường cao kẻ từ đỉnh $A.$ Gọi $M$ là trung điểm của $BC$. Gọi $Q$ là điểm nằm trên $\Gamma $sao cho $\angle HQA=90^\circ $và gọi $K$ là điểm nằm trên $\Gamma $sao cho $\angle HKQ=90^\circ $. Giả sử các điểm $A,B,C,K$và $Q$ đều phân biệt và nằm trên $\Gamma $ theo thứ tự đó.
Chứng minh rằng đường tròn ngoại tiếp hai tam giác $KQH$ và $FKM$ tiếp xúc nhau.
[RIGHT][I][B]Nguồn: MathScope.ORG[/B][/I][/RIGHT]
 
Hình Kèm Theo
Kiểu File : jpg 11206953_10153021777821094_1965956100181465054_n.jpg (89.0 KB, 116 lần tải)
__________________
Sự im lặng của bầy mèo

thay đổi nội dung bởi: huynhcongbang, 10-07-2015 lúc 04:17 PM
huynhcongbang is offline   Trả Lời Với Trích Dẫn
The Following 6 Users Say Thank You to huynhcongbang For This Useful Post:
Highschoolmath (10-07-2015), manhnguyen94 (10-07-2015), quocbaoct10 (10-07-2015), thaygiaocht (10-07-2015), thiendieu96 (18-08-2015), vantienducdh (10-07-2015)
Old 10-07-2015, 03:40 PM   #2
thaygiaocht
+Thành Viên+
 
thaygiaocht's Avatar
 
Tham gia ngày: Aug 2012
Đến từ: Chuyên Hà Tĩnh
Bài gởi: 165
Thanks: 793
Thanked 216 Times in 93 Posts
Câu 3 hình là lợi thế của Đoàn Việt Nam ta rồi.
Mấu chốt là có tứ giác điều hòa do đường thẳng đậm là tiếp tuyến của đường tròn đậm.
Cụ thể như sau:
Ta có $\widehat{JHK}=\widehat{AQ'H}=\widehat{HH'K}$ nên $HM$ là tiếp tuyến của $(KHH').$
Khi đó $\widehat{HKM}=\widehat{FKH'}.$
Biến đổi góc ta được $\widehat{JKM}+\widehat{FMK}=90^0,$ đây chính là đpcm.

[RIGHT][I][B]Nguồn: MathScope.ORG[/B][/I][/RIGHT]
 
__________________
https://www.facebook.com/thaygiaocht

thay đổi nội dung bởi: thaygiaocht, 10-07-2015 lúc 08:00 PM
thaygiaocht is offline   Trả Lời Với Trích Dẫn
The Following 3 Users Say Thank You to thaygiaocht For This Useful Post:
Highschoolmath (10-07-2015), quocbaoct10 (10-07-2015), Raul Chavez (11-07-2015)
Old 10-07-2015, 06:28 PM   #3
quocbaoct10
+Thành Viên Danh Dự+
 
quocbaoct10's Avatar
 
Tham gia ngày: Oct 2012
Đến từ: THPT chuyên Lê Quý Đôn-Nha Trang-Khánh Hòa
Bài gởi: 539
Thanks: 292
Thanked 365 Times in 217 Posts
Câu 1:
Bổ đề: các đa giác đều có số lẻ cạnh đều là tập "cân bằng" và "không tâm".
Chứng minh: Gọi đa giác đều trên là $A_1A_2...A_{2k+1}$. Gọi $(O)$ là đường tròn ngoại tiếp đa giác đều trên. Gọi đường kính đi qua tâm và 1 đỉnh của đa giác đều trên là "trục" của đa giác đều thì ta luôn có $2k+1$ trục như vậy. xét các cạnh $A_{i}A_1,A_{i}A_2,...,A_{i}A_{i-1},A_{i+1},...,A_{i}A_{2i+1}$, vì qua $A_{i}$ có 1 trục của đa giác đều nên trong số $2i$ cạnh kia có thể chia thành $i$ cặp cạnh bằng nhau với các độ dài phân biệt không bằng nhau. Từ đấy suy ra không tồn tại 3 cạnh chung đỉnh nào của đa giác đều bằng nhau Vậy ta chứng minh được bổ đề.
a). Từ các $2k+1$-giác đều, ta dựng tâm của các đa giác đấy. Khi đó, ta được một tập "cân bằng" có số chẵn điểm. Kết hợp với các $2k+1$-giác đều, ta có đpcm.
b). Xét 1 tập cân bằng $A$ có $2k$ điểm $A_1,A_2,A_3,...A_{2k}$. 1 điểm $A_i$ gọi là tốt đối với 1 cặp $A_{k},A_{t}$ ($k \neq i \neq t $) nếu $A_{i}A_{k}=A_{i}A_{t}$. Có tất cả $C_{2k}^{2}=k(2k-1)$ cặp điểm, mà trong $S$ có $2k$ điểm nên theo Dirichlet thì phải tồn tại 1 điểm $A_i$ tốt với $k$ cặp điểm. Nhưng vì trong $k$ cặp này không có cặp chứa điểm $A_i$ nên sẽ phải có 2 cặp $(A_t,A_{t_1})$ và $(A_t,A_{t_2})$ có trùng 1 điểm $A_t$.Khi đó ta có $A_i=A_{t_1}=A_{t_2}$, suy ra tập $S$ có $2k$ phần tử không phải là tập không tâm. Kết hợp với bổ đề, với các số tự nhiên $n$ lẻ lớn hơn $1$ thì luôn tồn tại $S$ có $n$ phần tử sao cho tồn tại một tập vừa cân bằng, vừa không tâm và có $n$ điểm.
[RIGHT][I][B]Nguồn: MathScope.ORG[/B][/I][/RIGHT]
 
__________________
i'll try my best.
quocbaoct10 is offline   Trả Lời Với Trích Dẫn
The Following 2 Users Say Thank You to quocbaoct10 For This Useful Post:
Highschoolmath (10-07-2015), thaygiaocht (10-07-2015)
Old 10-07-2015, 07:37 PM   #4
Traum
Moderator
 
Traum's Avatar
 
Tham gia ngày: Nov 2007
Đến từ: cyber world
Bài gởi: 413
Thanks: 14
Thanked 466 Times in 171 Posts
Trích:
Nguyên văn bởi quocbaoct10 View Post
Câu 1:
Bổ đề: các đa giác đều có số lẻ cạnh đều là tập "cân bằng" và "không tâm".
Chứng minh: Gọi đa giác đều trên là $A_1A_2...A_{2k+1}$. Gọi $(O)$ là đường tròn ngoại tiếp đa giác đều trên. Gọi đường kính đi qua tâm và 1 đỉnh của đa giác đều trên là "trục" của đa giác đều thì ta luôn có $2k+1$ trục như vậy. xét các cạnh $A_{i}A_1,A_{i}A_2,...,A_{i}A_{i-1},A_{i+1},...,A_{i}A_{2i+1}$, vì qua $A_{i}$ có 1 trục của đa giác đều nên trong số $2i$ cạnh kia có thể chia thành $i$ cặp cạnh bằng nhau với các độ dài phân biệt không bằng nhau. Từ đấy suy ra không tồn tại 3 cạnh chung đỉnh nào của đa giác đều bằng nhau Vậy ta chứng minh được bổ đề.
a). Từ các $2k+1$-giác đều, ta dựng tâm của các đa giác đấy. Khi đó, ta được một tập "cân bằng" có số chẵn điểm. Kết hợp với các $2k+1$-giác đều, ta có đpcm.
b). Xét 1 tập cân bằng $A$ có $2k$ điểm $A_1,A_2,A_3,...A_{2k}$. 1 điểm $A_i$ gọi là tốt đối với 1 cặp $A_{k},A_{t}$ ($k \neq i \neq t $) nếu $A_{i}A_{k}=A_{i}A_{t}$. Có tất cả $C_{2k}^{2}=k(2k-1)$ cặp điểm, mà trong $S$ có $2k$ điểm nên theo Dirichlet thì phải tồn tại 1 điểm $A_i$ tốt với $k$ cặp điểm. Nhưng vì trong $k$ cặp này không có cặp chứa điểm $A_i$ nên sẽ phải có 2 cặp $(A_t,A_{t_1})$ và $(A_t,A_{t_2})$ có trùng 1 điểm $A_t$.Khi đó ta có $A_i=A_{t_1}=A_{t_2}$, suy ra tập $S$ có $2k$ phần tử không phải là tập không tâm. Kết hợp với bổ đề, với các số tự nhiên $n$ lẻ lớn hơn $1$ thì luôn tồn tại $S$ có $n$ phần tử sao cho tồn tại một tập vừa cân bằng, vừa không tâm và có $n$ điểm.
Bổ đề và lời giải câu b chuẩn rồi. Tuy nhiên lời giải cho câu a ở chỗ thêm tâm chưa chuẩn.

Nếu gọi đa giác là $A_1,A_2,..,A_{2k+1}$ và tâm là $O$ thì có thể không tồn tại $C$ mà $C$ cách đều $O$ và $A_1$.

Cách chỉ ra cho $n=2k$ chẵn là: với $n=4$, dựng 2 tam giác đều cạnh đơn vị $OA_1A_2$ và $OA_2A_3$. Từ $n$ lên $n+2$ thì chỉ việc dựng thêm tam giác đều cạnh đơn vị $OA_{n}A_{n+1}$.

Dễ thấy với mọi $A_i\neq A_j$ thì $OA_i = OA_j = 1$. Với mọi $A_i$ với $O$, xét tam giác đều tương ứng ta có $A_j$ mà $OA_j = A_iA_j = 1$
[RIGHT][I][B]Nguồn: MathScope.ORG[/B][/I][/RIGHT]
 
__________________
Traum is giấc mơ.

thay đổi nội dung bởi: Traum, 10-07-2015 lúc 09:00 PM
Traum is offline   Trả Lời Với Trích Dẫn
The Following User Says Thank You to Traum For This Useful Post:
quocbaoct10 (10-07-2015)
Old 10-07-2015, 07:59 PM   #5
tr minh
+Thành Viên+
 
Tham gia ngày: Jul 2015
Bài gởi: 1
Thanks: 0
Thanked 0 Times in 0 Posts
Mình đã gửi lời giải bài 2 nhưng ko gõ latex kịp
[RIGHT][I][B]Nguồn: MathScope.ORG[/B][/I][/RIGHT]
 
tr minh is offline   Trả Lời Với Trích Dẫn
Old 10-07-2015, 08:03 PM   #6
Highschoolmath
Moderator
 
Highschoolmath's Avatar
 
Tham gia ngày: Apr 2008
Đến từ: Hàm Dương-Đại Tần
Bài gởi: 698
Thanks: 247
Thanked 350 Times in 224 Posts
Trích:
Nguyên văn bởi Traum View Post
Bổ đề và lời giải câu b chuẩn rồi. Tuy nhiên lời giải cho câu a ở chỗ thêm tâm chưa chuẩn.

Nếu gọi đa giác là $A_1,A_2,..,A_{2k+1}$ và tâm là $O$ thì có thể không tồn tại $C$ mà $C$ cách đều $O$ và $A_1$.

Cách chỉ ra cho $n=2k$ chẵn là: với $n=4$, dựng 2 tam giác đều cạnh đơn vị $OA_1A_2$ và $OA_2A_3$. Từ $n$ lên $n+2$ thì chỉ việc dựng thêm tam giác đều cạnh đơn vị $OA_{n}OA_{n+1}$.

Dễ thấy với mọi $A_i\neq A_j$ thì $OA_i = OA_j = 1$. Với mọi $A_i$ với $O$, xét tam giác đều tương ứng ta có $A_j$ mà $OA_j = A_iA_j = 1$
Câu 1a có thể dựng thế này được không anh Giấc Mơ nhỉ?
Xét hệ tọa độ xiên xOy với góc xOy bằng 60 độ. Dựng các điểm theo trình tự (0,0), (1,0), (0,1), (1,1), (2,0), (0,2),....,(1,k-1), (2,k-2),...,(k-1,1), (0,k), (k,0). Với mỗi n thì chỉ cần lấy n điểm đầu tiên trong dãy điểm này là được.
[RIGHT][I][B]Nguồn: MathScope.ORG[/B][/I][/RIGHT]
 
__________________
As long as I live, I shall think only of the Victory......................
Highschoolmath is offline   Trả Lời Với Trích Dẫn
Old 10-07-2015, 08:18 PM   #7
Traum
Moderator
 
Traum's Avatar
 
Tham gia ngày: Nov 2007
Đến từ: cyber world
Bài gởi: 413
Thanks: 14
Thanked 466 Times in 171 Posts
Trích:
Nguyên văn bởi Highschoolmath View Post
Câu 1a có thể dựng thế này được không anh Giấc Mơ nhỉ?
Xét hệ tọa độ xiên xOy với góc xOy bằng 60 độ. Dựng các điểm theo trình tự (0,0), (1,0), (0,1), (1,1), (2,0), (0,2),....,(1,k-1), (2,k-2),...,(k-1,1), (0,k), (k,0). Với mỗi n thì chỉ cần lấy n điểm đầu tiên trong dãy điểm này là được.
Chắc là ko. Xét cặp (1,0) và (0,3) xem?
[RIGHT][I][B]Nguồn: MathScope.ORG[/B][/I][/RIGHT]
 
__________________
Traum is giấc mơ.
Traum is offline   Trả Lời Với Trích Dẫn
The Following User Says Thank You to Traum For This Useful Post:
Highschoolmath (10-07-2015)
Old 10-07-2015, 08:57 PM   #8
quocbaoct10
+Thành Viên Danh Dự+
 
quocbaoct10's Avatar
 
Tham gia ngày: Oct 2012
Đến từ: THPT chuyên Lê Quý Đôn-Nha Trang-Khánh Hòa
Bài gởi: 539
Thanks: 292
Thanked 365 Times in 217 Posts
Câu 2: Theo đề bài, ta đặt $ab-c=2^x, bc-a=2^y,ca-b=2^z$ , giả sử $x \ge y\ ge z$, từ đó ta có $b \ge a \ge c$. Với $y=z$ dễ dàng tìm ra bộ nghiệm $(2,2,2)$. Xét TH $y>z$, giả sử $c > 4$ .
Đầu tiên, ta sẽ chứng minh $3a < b$. Giả sử như $3a > b $ thì $-3a<-b \Leftrightarrow a(c-3) < 2^{z}$ hay $a < 2^{z-1}$.
Ta có:
$\begin{cases}ab-c-bc+a=2^z(2^{x-y}-1)\\ab-c+bc-a=2^z(2^{x+y}+1)\end{cases} \\
\Leftrightarrow \begin{cases}(b+1)(a-c)=2^y(2^{x-y}-1) (1)\\(b-1)(c+a)=2^y(2^{x-y}+1)(2) \end{cases}$(*)
Từ (*) ta thấy nếu $b+1$ chia hết cho 4 thì $b-1$ không chia hết cho 4, hay $c+a$ chia hết cho $2^{y-1}$ hay $a \ge 2^{y-2} \ge 2^{z-1}$ (vô lý). Tương tự như vậy với trường hợp $b-1$ chia hết cho 4 và trường hợp cả $b-1$ lẫn $b+1$ không chia hết cho 4, ta được $3a<b$.

Có:
$\begin{cases}bc-a-ca+b=2^z(2^{y-z}-1)\\bc-a+ca-b=2^z(2^{y-z}+1)\end{cases} \\
\Leftrightarrow \begin{cases}(c+1)(b-a)=2^z(2^{y-z}-1) (1)\\(c-1)(b+a)=2^z(2^{y-z}+1)(2) \end{cases}$ .
Lấy (1) chia (2), được:
$\frac{b-a}{b+a}=\frac{2^{y-z}-1}{2^{y-z}+1}.\frac{c-1}{c+1} \ge \frac{2^{y-z}-1}{2^{y-z}+1}.\frac{1}{2} \\
\Leftrightarrow 2^{y-z} \le \frac{3b-a}{3a-b}$.
Mà $3a-b < 0$ nên $2^{y-z} < 0$ (vô lý). Vậy nên $c \le 4$.
Từ đó tìm ra được các bộ nghiệm ứng với $(a,b,c)$ là $(2,2,2), (2,3,2), (5,7,3), (6,11,2)$.
[RIGHT][I][B]Nguồn: MathScope.ORG[/B][/I][/RIGHT]
 
__________________
i'll try my best.

thay đổi nội dung bởi: quocbaoct10, 16-07-2015 lúc 02:13 PM
quocbaoct10 is offline   Trả Lời Với Trích Dẫn
The Following 4 Users Say Thank You to quocbaoct10 For This Useful Post:
dangnamneu (11-07-2015), pco (11-07-2015), sieusieu90 (10-07-2015), thaygiaocht (10-07-2015)
Old 11-07-2015, 07:32 AM   #9
truongson2007
Super Moderator
 
Tham gia ngày: Jan 2008
Bài gởi: 53
Thanks: 109
Thanked 27 Times in 17 Posts
Gọi W, N, P lần lượt là trung điểm của HA,HQ,HK. T là điểm đối xứng của Q qua W. Dễ thấy HT là tiếp tuyến của đường tròn (N), (PNH),(HFM). Sử dụng phép vị tự tâm H tỉ số 1/2 dễ dàng suy ra P,N,W nằm trện đường tròn Euler. Suy ra HT, FM, NP đồng quy tại J. Việc còn lại là chứng minh JK là tiếp tuyen (N). Suy ra đpcm
[RIGHT][I][B]Nguồn: MathScope.ORG[/B][/I][/RIGHT]
 

thay đổi nội dung bởi: truongson2007, 11-07-2015 lúc 08:51 AM
truongson2007 is offline   Trả Lời Với Trích Dẫn
Old 11-07-2015, 10:50 AM   #10
mchuy
+Thành Viên+
 
Tham gia ngày: Sep 2009
Bài gởi: 22
Thanks: 6
Thanked 7 Times in 5 Posts
Trích:
Nguyên văn bởi Traum View Post
Bổ đề và lời giải câu b chuẩn rồi. Tuy nhiên lời giải cho câu a ở chỗ thêm tâm chưa chuẩn.

Nếu gọi đa giác là $A_1,A_2,..,A_{2k+1}$ và tâm là $O$ thì có thể không tồn tại $C$ mà $C$ cách đều $O$ và $A_1$.

Cách chỉ ra cho $n=2k$ chẵn là: với $n=4$, dựng 2 tam giác đều cạnh đơn vị $OA_1A_2$ và $OA_2A_3$. Từ $n$ lên $n+2$ thì chỉ việc dựng thêm tam giác đều cạnh đơn vị $OA_{n}A_{n+1}$.

Dễ thấy với mọi $A_i\neq A_j$ thì $OA_i = OA_j = 1$. Với mọi $A_i$ với $O$, xét tam giác đều tương ứng ta có $A_j$ mà $OA_j = A_iA_j = 1$
Nếu dựng như vậy khi n bằng 8 thì điểm A_7 trùng lại A_1 đâu còn đúng nữa anh
[RIGHT][I][B]Nguồn: MathScope.ORG[/B][/I][/RIGHT]
 
mchuy is offline   Trả Lời Với Trích Dẫn
Old 11-07-2015, 12:38 PM   #11
vanchay
+Thành Viên+
 
Tham gia ngày: May 2012
Bài gởi: 20
Thanks: 68
Thanked 21 Times in 12 Posts
Em gửi bản dịch tạm đề ngày thi thứ 2.
[RIGHT][I][B]Nguồn: MathScope.ORG[/B][/I][/RIGHT]
 
Hình Kèm Theo
Kiểu File : jpg Untitled.jpg (43.8 KB, 124 lần tải)
Kiểu File : png Untitled1.png (73.8 KB, 85 lần tải)
Kiểu File : jpg Untitled13.jpg (28.0 KB, 191 lần tải)

thay đổi nội dung bởi: vanchay, 11-07-2015 lúc 12:42 PM
vanchay is offline   Trả Lời Với Trích Dẫn
Old 11-07-2015, 01:05 PM   #12
thaygiaocht
+Thành Viên+
 
thaygiaocht's Avatar
 
Tham gia ngày: Aug 2012
Đến từ: Chuyên Hà Tĩnh
Bài gởi: 165
Thanks: 793
Thanked 216 Times in 93 Posts
Trích:
Nguyên văn bởi vanchay View Post
Em gửi bản dịch tạm đề ngày thi thứ 2.
Lại có hình và PTH là 2 phần thế mạnh của VN nên khả năng có nhiều vàng rồi.
Bài hình ngày 2 cơ bản cũng là một bài toán biến đổi góc.
Cụ thể như sau:
Đpcm tương đương với $YF=YG$ hay $\widehat{AFO_1}=\widehat{AGO_2}$ hay $\widehat{BFD}=\widehat{CGE}.$
Điều này đúng theo cách xác định các điểm $F,D,E,G.$ Thật vậy
$\widehat{DAF}+\widehat{DBF}=\widehat{C}+\widehat{ \alpha}$ và $\widehat{ECG}+\widehat{EAG}=\widehat{B}+\widehat{ \alpha}.$
Trừ từng vế ta được $\widehat{AGE}-\widehat{ADF}=\widehat{C}-\widehat{B}.$
Chuyển vế ta được $\widehat{BFD}=\widehat{CGE},$ đpcm.

[RIGHT][I][B]Nguồn: MathScope.ORG[/B][/I][/RIGHT]
 
__________________
https://www.facebook.com/thaygiaocht

thay đổi nội dung bởi: thaygiaocht, 11-07-2015 lúc 02:21 PM
thaygiaocht is offline   Trả Lời Với Trích Dẫn
Old 11-07-2015, 01:59 PM   #13
hansongkyung
+Thành Viên+
 
Tham gia ngày: Jan 2012
Đến từ: Han Tae Woong - IMO 1998
Bài gởi: 493
Thanks: 109
Thanked 417 Times in 241 Posts
Gửi tin nhắn qua Yahoo chát tới hansongkyung
Các bạn cho mình hỏi là hàm số nếu chỉ cho điều kiện từ R -> R thì mình có thể cho x chạy đến vô cùng để tìm hàm được không? Mình có nghĩ ra một cách để giải bài 2 ngày 2 mà mông lung phần kia quá.
[RIGHT][I][B]Nguồn: MathScope.ORG[/B][/I][/RIGHT]
 
hansongkyung is offline   Trả Lời Với Trích Dẫn
Old 11-07-2015, 07:02 PM   #14
quocbaoct10
+Thành Viên Danh Dự+
 
quocbaoct10's Avatar
 
Tham gia ngày: Oct 2012
Đến từ: THPT chuyên Lê Quý Đôn-Nha Trang-Khánh Hòa
Bài gởi: 539
Thanks: 292
Thanked 365 Times in 217 Posts
Câu 5:
CHo $x=y=0$ thì được $f(f(0))=f(0)$.
Thay $x=0, y=f(0)$, ta có $2f(0)=f(0)^2 \Rightarrow f(0)=0$ hoặc $f(0)=2$.
TH1: $f(0)=2$.
Thay $y$ bởi $x$, $x=0$ vào pt đầu, ta được: $f(f(x))=f(x)+2(x-1)$ (1)
Thay $x$ bởi $x-1$, $y=1$ vào pt đầu, ta được: $f(x+f(x)-1)=x+f(x)-1$
Thay $x$ bởi $x+f(x)-1$ vào (1), ta được: $f(x)=2-x$.
TH2: f(0)=0.
Giả sử $f(x) \neq x$ $\forall x \in \mathbb{R}$.
Thay $y$ bởi $f(x)-x$, ta được: $f(x(f(x)-x))=(f(x)-x)f(x)$ (2)
Thay $y$ bởi $x-f(x)$, $x$ bởi $-x$, ta được: $f(-x(x-f(x)))=(x-f(x))f(-x)$ (3)
Từ (3) và (2) suy ra $f(x)=-f(-x)$ (*)
Thay $y$ bởi $-x$ vào pt đầu, ta được: $f(x)+f(-x^2)=x-xf(x)$ (4)
Thay $x$ bởi $-x$ vào pt (4), ta được: $f(-x)+f(-x^2)=-x+x(f-x)$ (5)
Từ (4) và (5) suy ra: $-f(x)+x-xf(x)=-f(-x)-x+xf(-x)$ (6)
Thay (*) vào (6), ta có: $-f(x)+x-xf(x)=f(x)-x-xf(x) \Leftrightarrow f(x)=x$ (vô lý).
Vậy: $f(x)=x$ $\forall x \in \mathbb{R}$.
Vậy nghiệm của pt hàm là : $f(x)=2-x$ và $f(x)=x$.
[RIGHT][I][B]Nguồn: MathScope.ORG[/B][/I][/RIGHT]
 
__________________
i'll try my best.

thay đổi nội dung bởi: quocbaoct10, 11-07-2015 lúc 07:04 PM
quocbaoct10 is offline   Trả Lời Với Trích Dẫn
Old 13-07-2015, 04:06 AM   #15
Kelacloi
+Thành Viên+
 
Tham gia ngày: Mar 2011
Bài gởi: 252
Thanks: 50
Thanked 164 Times in 114 Posts
Bài 6 rất khó ưa.
Nếu chặn là $2014^2$ thì bài toán sẽ rất dễ thương, nhưng lấy chặn $1007^2$ thì khiến người ăn k ngon.

Bài này viết dưới dạng bảng có lẽ dễ nhìn hơn.

Chuyển về bảng trên bảng
Ta xây dựng bảng ô vuông có chiều dài chiều rộng tuỳ ý như sau.
Nếu $a_n=k$ thì các ô $(n,n)$ đến $(n+k-1,n)$ được điền số 1.

Ta gọi ô $(n+k-1,n)$ là ô thấp nhất của cột $n$.
Điều kiện của bài toán sẽ trở thành như sau:
i) Tổng các số trên mỗi cột lớn hơn $0$ và nhỏ hơn $2015$
ii) $2$ ô thấp nhất của $2$ cột bất kì không nằm chung 1 hàng.

Chọn $N,b$

Xét đơn biến $S(n)$ là tổng các số trên hàng $n$.
Ta sẽ chứng minh được :$S(n)$ không giảm và $S(n) \le 2015$.
Bời thế tồn tại $N,b \in \mathbb{N}$ sao cho : $S(n)=b \forall n>N$. (*)

Ta chọn $N,b$ cho bất đẳng thức cần chứng minh như $N,b$ được kiếm ra ở trên.

Giải nghĩa vế trái BDT theo bảng
Với $n>m \ge N$.

Đặt :
$A$ là tổng các số từ cột $m+1$ đến cột $n$
$B$ là tổng các số từ hàng $m+1$ đến cột $n$
$C$ là tổng các số nằm trong hình chữ nhật $ (m+1,m+1) : (n,n)$

Ta có :
1)$A= \sum_{k=m+1}^n a_k$
2)$A-C$ là tổng các số nằm có cột nằm từ $(m+1)$ đến $n$ và nằm từ hàng $n+1$ trở đi.
3) $B= \sum_{k=m+1}^n b $ (do (*))
4) $B-C$ là tổng các số nằm có hàng nằm từ $(m+1)$ đến $n$ và nằm từ cột $m$ trở xuống.

BDT ta cần chứng minh thực ra là $| (A-C)-(B-C) | \le 1007^2$

Về bất đẳng thức
Giờ ta thấy ngay vầy, nếu đánh giá "nhẹ nhàng" thì ta có :
$ 0 \le A-C,B-C \le 2014^2$
bởi thế nên ta sẽ có ngay $|A-B| \le 2014^2$ mà không quá lo lắng .

Tiếc thay đề bài lại là $1007^2$.
Với $n-m \ge 2015$, ,mình tạm đánh giá bằng trung gian mà chẳng "nhẹ nhàng" để diễn đạt lắm, thì được cái sau đây:
$ \frac{b(b+1)}{2} \le B-C \le (b-1)(2015-\frac{b}{2})$
$ \frac{b(b-1)}{2} \le A-C \le b(2015-\frac{b+1}{2})$
Từ đó ta được $ -(b-1)(2015-b) \le A-B \le b(2014-b)$

Trích:
Để mình ghi chi tiết cách đánh giá 1 trong các bdt phía trên:
$ A-C \le b(2015-\frac{b+1}{2})$

Ta có $S(n+1)=b$, thế nên chỉ có đúng $b$ cột trong $(m+1) \rightarrow n+1$ là có ô được điền số 1 nằm trong " cột nằm từ $(m+1)$ đến $n+1$ và từ hàng $n+1$ trở đi"
hay :
chỉ có đúng $b-1$ cột trong $(m+1) \rightarrow n$ là có ô được điền số 1 nằm trong " cột nằm từ $(m+1)$ đến $n$ và từ hàng $n+1$ trở đi" (tức $(A-C)$) (1)

Ta trích 1 bảng mới tạo bởi "cột nằm từ $(m+1)$ đến $n$" và "từ hàng $n+1$ trở đi."
Do :" $2$ ô thấp nhất của $2$ cột bất kì không nằm chung 1 hàng"
nên 2 cột bất kì trong bảng mới có tổng khác nhau. (2)

Do " $a_k \le 2015$ "
nên tổng mỗi tổng mỗi cột trong bảng mới không đều không quá $2014$.(3)

Từ (1),(2),(3), ta được:
$A-C \le 2014+(2014-1)+...(2014-b+2) < b(2015-\frac{b+1}{2})$

Đến đây thì rút được ngay kết luận.

Với $n-m \le 2014$ thì cũng thế nhưng nhìn rối mắt :v
[RIGHT][I][B]Nguồn: MathScope.ORG[/B][/I][/RIGHT]
 
__________________

thay đổi nội dung bởi: Kelacloi, 13-07-2015 lúc 09:47 PM
Kelacloi is offline   Trả Lời Với Trích Dẫn
The Following 3 Users Say Thank You to Kelacloi For This Useful Post:
kinhluannguyen (15-07-2015), quocbaoct10 (13-07-2015), vinhhop.qt (13-07-2015)
Trả lời Gởi Ðề Tài Mới

Bookmarks

Ðiều Chỉnh
Xếp Bài

Quuyền Hạn Của Bạn
You may not post new threads
You may not post replies
You may not post attachments
You may not edit your posts

BB code is Mở
Smilies đang Mở
[IMG] đang Mở
HTML đang Tắt

Chuyển đến


Múi giờ GMT. Hiện tại là 12:48 PM.


Powered by: vBulletin Copyright ©2000-2024, Jelsoft Enterprises Ltd.
Inactive Reminders By mathscope.org
[page compression: 114.31 k/131.19 k (12.87%)]